Show that for M symmetric positive definite have $sup_{||x||=1}langle x, Mx rangle$=$lambda_1=:$ the largest...











up vote
0
down vote

favorite












I know that if I can reach the step
$$sup_{||x||=1}langle x, Mx rangle=langle x,lambda_1 xrangle$$ then clearly $lambda_1$ follows. But how can we bridge this gap?










share|cite|improve this question




















  • 1




    The key word to search is "Rayleigh quotient". See also: people.math.gatech.edu/~ecroot/notes_linear.pdf
    – user587192
    20 hours ago












  • Or alternatively, consider $varphi(t) = f(x + ty)$, where $f(x) = left(Mxmiddle|xright)/(x|x)$. Then $varphi(t)$ attains maximum at $t=0$, thus the differential calculus would work.
    – xbh
    20 hours ago















up vote
0
down vote

favorite












I know that if I can reach the step
$$sup_{||x||=1}langle x, Mx rangle=langle x,lambda_1 xrangle$$ then clearly $lambda_1$ follows. But how can we bridge this gap?










share|cite|improve this question




















  • 1




    The key word to search is "Rayleigh quotient". See also: people.math.gatech.edu/~ecroot/notes_linear.pdf
    – user587192
    20 hours ago












  • Or alternatively, consider $varphi(t) = f(x + ty)$, where $f(x) = left(Mxmiddle|xright)/(x|x)$. Then $varphi(t)$ attains maximum at $t=0$, thus the differential calculus would work.
    – xbh
    20 hours ago













up vote
0
down vote

favorite









up vote
0
down vote

favorite











I know that if I can reach the step
$$sup_{||x||=1}langle x, Mx rangle=langle x,lambda_1 xrangle$$ then clearly $lambda_1$ follows. But how can we bridge this gap?










share|cite|improve this question















I know that if I can reach the step
$$sup_{||x||=1}langle x, Mx rangle=langle x,lambda_1 xrangle$$ then clearly $lambda_1$ follows. But how can we bridge this gap?







linear-algebra eigenvalues-eigenvectors






share|cite|improve this question















share|cite|improve this question













share|cite|improve this question




share|cite|improve this question








edited 19 hours ago









user587192

98410




98410










asked 20 hours ago









Dan

827




827








  • 1




    The key word to search is "Rayleigh quotient". See also: people.math.gatech.edu/~ecroot/notes_linear.pdf
    – user587192
    20 hours ago












  • Or alternatively, consider $varphi(t) = f(x + ty)$, where $f(x) = left(Mxmiddle|xright)/(x|x)$. Then $varphi(t)$ attains maximum at $t=0$, thus the differential calculus would work.
    – xbh
    20 hours ago














  • 1




    The key word to search is "Rayleigh quotient". See also: people.math.gatech.edu/~ecroot/notes_linear.pdf
    – user587192
    20 hours ago












  • Or alternatively, consider $varphi(t) = f(x + ty)$, where $f(x) = left(Mxmiddle|xright)/(x|x)$. Then $varphi(t)$ attains maximum at $t=0$, thus the differential calculus would work.
    – xbh
    20 hours ago








1




1




The key word to search is "Rayleigh quotient". See also: people.math.gatech.edu/~ecroot/notes_linear.pdf
– user587192
20 hours ago






The key word to search is "Rayleigh quotient". See also: people.math.gatech.edu/~ecroot/notes_linear.pdf
– user587192
20 hours ago














Or alternatively, consider $varphi(t) = f(x + ty)$, where $f(x) = left(Mxmiddle|xright)/(x|x)$. Then $varphi(t)$ attains maximum at $t=0$, thus the differential calculus would work.
– xbh
20 hours ago




Or alternatively, consider $varphi(t) = f(x + ty)$, where $f(x) = left(Mxmiddle|xright)/(x|x)$. Then $varphi(t)$ attains maximum at $t=0$, thus the differential calculus would work.
– xbh
20 hours ago










1 Answer
1






active

oldest

votes

















up vote
0
down vote



accepted










By the spectral theorem we can take an $textit{orthonormal basis}$ of $mathbb R^n$ consisting of eigenvectors of $M$, say $v_1,ldots,v_n$ with corresponding eigenvalues $lambda_1,ldots,lambda_n$. Then, for any $Vert vVert=1$ we can write $v=c_1v_1+cdots+c_nv_n$. Try expanding $langle Mv,vrangle$ using linearity of the inner product, the fact that $langle v_i,v_jrangle=delta_{i,j}$, and the fact that $Mv_i=lambda_iv_i$ to show that $langle Mv,vrangleleqlambda^*$ where $lambda^*$ is your max eigenvalue. Then can you exhibit a specific $v$ with norm $1$ for which equality holds (think eigenvectors)?






share|cite|improve this answer

















  • 1




    Thank you. The orthogonal basis was the key. $||v||=1 Rightarrow sum_{i}^{n} c_i^2=1$ then write $langle v, Mv rangle= sum_i^n lambda_i c_i^2 leq sum_i^n lambda_1 c_i^2 = lambda_1$
    – Dan
    19 hours ago













Your Answer





StackExchange.ifUsing("editor", function () {
return StackExchange.using("mathjaxEditing", function () {
StackExchange.MarkdownEditor.creationCallbacks.add(function (editor, postfix) {
StackExchange.mathjaxEditing.prepareWmdForMathJax(editor, postfix, [["$", "$"], ["\\(","\\)"]]);
});
});
}, "mathjax-editing");

StackExchange.ready(function() {
var channelOptions = {
tags: "".split(" "),
id: "69"
};
initTagRenderer("".split(" "), "".split(" "), channelOptions);

StackExchange.using("externalEditor", function() {
// Have to fire editor after snippets, if snippets enabled
if (StackExchange.settings.snippets.snippetsEnabled) {
StackExchange.using("snippets", function() {
createEditor();
});
}
else {
createEditor();
}
});

function createEditor() {
StackExchange.prepareEditor({
heartbeatType: 'answer',
convertImagesToLinks: true,
noModals: true,
showLowRepImageUploadWarning: true,
reputationToPostImages: 10,
bindNavPrevention: true,
postfix: "",
imageUploader: {
brandingHtml: "Powered by u003ca class="icon-imgur-white" href="https://imgur.com/"u003eu003c/au003e",
contentPolicyHtml: "User contributions licensed under u003ca href="https://creativecommons.org/licenses/by-sa/3.0/"u003ecc by-sa 3.0 with attribution requiredu003c/au003e u003ca href="https://stackoverflow.com/legal/content-policy"u003e(content policy)u003c/au003e",
allowUrls: true
},
noCode: true, onDemand: true,
discardSelector: ".discard-answer"
,immediatelyShowMarkdownHelp:true
});


}
});














 

draft saved


draft discarded


















StackExchange.ready(
function () {
StackExchange.openid.initPostLogin('.new-post-login', 'https%3a%2f%2fmath.stackexchange.com%2fquestions%2f2995368%2fshow-that-for-m-symmetric-positive-definite-have-sup-x-1-langle-x-mx-r%23new-answer', 'question_page');
}
);

Post as a guest
































1 Answer
1






active

oldest

votes








1 Answer
1






active

oldest

votes









active

oldest

votes






active

oldest

votes








up vote
0
down vote



accepted










By the spectral theorem we can take an $textit{orthonormal basis}$ of $mathbb R^n$ consisting of eigenvectors of $M$, say $v_1,ldots,v_n$ with corresponding eigenvalues $lambda_1,ldots,lambda_n$. Then, for any $Vert vVert=1$ we can write $v=c_1v_1+cdots+c_nv_n$. Try expanding $langle Mv,vrangle$ using linearity of the inner product, the fact that $langle v_i,v_jrangle=delta_{i,j}$, and the fact that $Mv_i=lambda_iv_i$ to show that $langle Mv,vrangleleqlambda^*$ where $lambda^*$ is your max eigenvalue. Then can you exhibit a specific $v$ with norm $1$ for which equality holds (think eigenvectors)?






share|cite|improve this answer

















  • 1




    Thank you. The orthogonal basis was the key. $||v||=1 Rightarrow sum_{i}^{n} c_i^2=1$ then write $langle v, Mv rangle= sum_i^n lambda_i c_i^2 leq sum_i^n lambda_1 c_i^2 = lambda_1$
    – Dan
    19 hours ago

















up vote
0
down vote



accepted










By the spectral theorem we can take an $textit{orthonormal basis}$ of $mathbb R^n$ consisting of eigenvectors of $M$, say $v_1,ldots,v_n$ with corresponding eigenvalues $lambda_1,ldots,lambda_n$. Then, for any $Vert vVert=1$ we can write $v=c_1v_1+cdots+c_nv_n$. Try expanding $langle Mv,vrangle$ using linearity of the inner product, the fact that $langle v_i,v_jrangle=delta_{i,j}$, and the fact that $Mv_i=lambda_iv_i$ to show that $langle Mv,vrangleleqlambda^*$ where $lambda^*$ is your max eigenvalue. Then can you exhibit a specific $v$ with norm $1$ for which equality holds (think eigenvectors)?






share|cite|improve this answer

















  • 1




    Thank you. The orthogonal basis was the key. $||v||=1 Rightarrow sum_{i}^{n} c_i^2=1$ then write $langle v, Mv rangle= sum_i^n lambda_i c_i^2 leq sum_i^n lambda_1 c_i^2 = lambda_1$
    – Dan
    19 hours ago















up vote
0
down vote



accepted







up vote
0
down vote



accepted






By the spectral theorem we can take an $textit{orthonormal basis}$ of $mathbb R^n$ consisting of eigenvectors of $M$, say $v_1,ldots,v_n$ with corresponding eigenvalues $lambda_1,ldots,lambda_n$. Then, for any $Vert vVert=1$ we can write $v=c_1v_1+cdots+c_nv_n$. Try expanding $langle Mv,vrangle$ using linearity of the inner product, the fact that $langle v_i,v_jrangle=delta_{i,j}$, and the fact that $Mv_i=lambda_iv_i$ to show that $langle Mv,vrangleleqlambda^*$ where $lambda^*$ is your max eigenvalue. Then can you exhibit a specific $v$ with norm $1$ for which equality holds (think eigenvectors)?






share|cite|improve this answer












By the spectral theorem we can take an $textit{orthonormal basis}$ of $mathbb R^n$ consisting of eigenvectors of $M$, say $v_1,ldots,v_n$ with corresponding eigenvalues $lambda_1,ldots,lambda_n$. Then, for any $Vert vVert=1$ we can write $v=c_1v_1+cdots+c_nv_n$. Try expanding $langle Mv,vrangle$ using linearity of the inner product, the fact that $langle v_i,v_jrangle=delta_{i,j}$, and the fact that $Mv_i=lambda_iv_i$ to show that $langle Mv,vrangleleqlambda^*$ where $lambda^*$ is your max eigenvalue. Then can you exhibit a specific $v$ with norm $1$ for which equality holds (think eigenvectors)?







share|cite|improve this answer












share|cite|improve this answer



share|cite|improve this answer










answered 20 hours ago









Dave

8,16311033




8,16311033








  • 1




    Thank you. The orthogonal basis was the key. $||v||=1 Rightarrow sum_{i}^{n} c_i^2=1$ then write $langle v, Mv rangle= sum_i^n lambda_i c_i^2 leq sum_i^n lambda_1 c_i^2 = lambda_1$
    – Dan
    19 hours ago
















  • 1




    Thank you. The orthogonal basis was the key. $||v||=1 Rightarrow sum_{i}^{n} c_i^2=1$ then write $langle v, Mv rangle= sum_i^n lambda_i c_i^2 leq sum_i^n lambda_1 c_i^2 = lambda_1$
    – Dan
    19 hours ago










1




1




Thank you. The orthogonal basis was the key. $||v||=1 Rightarrow sum_{i}^{n} c_i^2=1$ then write $langle v, Mv rangle= sum_i^n lambda_i c_i^2 leq sum_i^n lambda_1 c_i^2 = lambda_1$
– Dan
19 hours ago






Thank you. The orthogonal basis was the key. $||v||=1 Rightarrow sum_{i}^{n} c_i^2=1$ then write $langle v, Mv rangle= sum_i^n lambda_i c_i^2 leq sum_i^n lambda_1 c_i^2 = lambda_1$
– Dan
19 hours ago




















 

draft saved


draft discarded



















































 


draft saved


draft discarded














StackExchange.ready(
function () {
StackExchange.openid.initPostLogin('.new-post-login', 'https%3a%2f%2fmath.stackexchange.com%2fquestions%2f2995368%2fshow-that-for-m-symmetric-positive-definite-have-sup-x-1-langle-x-mx-r%23new-answer', 'question_page');
}
);

Post as a guest




















































































Popular posts from this blog

Biblatex bibliography style without URLs when DOI exists (in Overleaf with Zotero bibliography)

ComboBox Display Member on multiple fields

Is it possible to collect Nectar points via Trainline?